Further explain argument
Hello- I had a very difficult time understanding the argument in itself, can you please break ...
@MichaelaJ on February 8 at 03:15AM
  • November 2018 LSAT
  • SEC1
  • Q16
1
Reply
Still need additional clarification
Hello- I am still not quite understanding why B is correct and D is incorrect. If we are looki...
@MichaelaJ on February 6 at 09:07PM
  • November 2018 LSAT
  • SEC1
  • Q12
2
Replies
C vs D
Hello Can you please explain what C & D are saying and how I can translate both of these to sc...
@MichaelaJ on February 6 at 08:57PM
  • November 2018 LSAT
  • SEC1
  • Q19
3
Replies
I hate these types of answer choices
The explanation for the correct answer was one of my two anticipations. But whenever the answer c...
AndrewArabie on May 26 at 06:52PM
  • November 2018 LSAT
  • SEC1
  • Q19
2
Replies
No Explanation Videos?
Uhhh, how come there are no explanation videos for this? This ain't new
Yuzhuo-Cao on March 19, 2023
  • November 2018 LSAT
  • SEC1
  • Q3
1
Reply
Why not D?
If industrial farming is more harmful to the environment than organic, then eventually it would s...
AndrewArabie on January 22, 2023
  • November 2018 LSAT
  • SEC1
  • Q21
3
Replies
Question Choice C
Wouldnt the elimination of the native species dependent make the gap wider between the statements...
the66guy on November 17, 2022
  • November 2018 LSAT
  • SEC1
  • Q13
2
Replies
Negate E please "few"?
Hi I negated "few" to "none," which halted my momentum and caused me to lose precious seconds!...
Mazen on July 22, 2022
  • November 2018 LSAT
  • SEC1
  • Q16
6
Replies
why is this wrong?
can some explain why this is wrong?
shonlumon on May 24, 2022
  • November 2018 LSAT
  • SEC1
  • Q24
3
Replies
premise
here i got caught up in the words "given that" as indicating a premise but the end of the sentenc...
akhan0102 on March 11, 2022
  • November 2018 LSAT
  • SEC1
  • Q4
1
Reply
Correct Answer
Please help explain why the answer is b and not d. Thank you
bb427 on February 27, 2022
  • November 2018 LSAT
  • SEC1
  • Q12
2
Replies
not sure why this is wrong.
can someone explain to me how to solve this problem?
shonlumon on February 8, 2022
  • November 2018 LSAT
  • SEC1
  • Q25
4
Replies
Weaken question
Can someone explain how C weakens this?
DalilaPando on January 19, 2022
  • November 2018 LSAT
  • SEC1
  • Q3
1
Reply
only if
Is this a good way to look at conditional logic? In a necessary assumption question and other...
c0cald01 on January 13, 2022
  • November 2018 LSAT
  • SEC1
  • Q5
2
Replies
Answer choice C
I did not understand what C meant. I thought it was irrelevant. Can you please explain?
amarachicynthia on May 3, 2021
  • November 2018 LSAT
  • SEC1
  • Q13
1
Reply
Help
Can someone please explain this question? Thanks!
Minerva on May 3, 2021
  • November 2018 LSAT
  • SEC1
  • Q13
3
Replies
Why e?
A C and D all seem to be better answers
Aidyn-Carlson on October 15, 2020
  • November 2018 LSAT
  • SEC1
  • Q17
5
Replies
terminology
Do the words " more likely than" signal proportionality?
YulissaCardoza on June 26, 2020
  • November 2018 LSAT
  • SEC1
  • Q12
2
Replies
Why B? Why not D?
Hey. I anticipated this answer that the farmer jumps to extreme. My issue here is why is D wrong....
avif on June 19, 2020
  • November 2018 LSAT
  • SEC1
  • Q21
2
Replies
Choice C
I understand why D is correct, however I thought that C would also be correct because the stimulu...
colleen_ on May 16, 2020
  • November 2018 LSAT
  • SEC1
  • Q8
1
Reply